Maratonas de FísicaV Maratona de Física IME/ITA

Moderador: [ Moderadores TTB ]

LPavaNNN
2 - Nerd
Mensagens: 140
Registrado em: Sáb 03 Nov, 2012 23:14
Última visita: 01-08-20
Jan 2017 28 13:43

Re: V Maratona de Física IME/ITA

Mensagem não lida por LPavaNNN »

Solução Problema 20:
O movimento em questão é um MHS e portanto tem período:

[tex3]T=2\pi\cdot \sqrt{\frac{k}{m}}[/tex3]

é necessário então, descobrir k

[tex3]F_G=k\cdot x[/tex3]

[tex3]\frac{GMm}{R^2}=kR[/tex3]

[tex3]k=\frac{GMm}{R^3}[/tex3]

substituindo a massa do planeta por volume x densidade:

[tex3]k=Gm.\frac{4\pi R^3}{3R^3}[/tex3]

[tex3]k=\frac{4\pi Gm}{3}[/tex3]

substituindo na equação do período

[tex3]T=2\pi.\sqrt{\frac{3}{4\pi pG}}[/tex3]

no entanto, o enunciado pede o tempo para atravessar o planeta, ou seja, metade do período:

[tex3]t=\pi.\sqrt{\frac{3}{4\pi p G}}[/tex3]

[tex3]t=\sqrt{\frac{3\pi}{4pG}}[/tex3]

Resposta B

-------------------------------------------------------------------------------------------

Problema 21 (IME-2015/2016)
IME.jpg
IME.jpg (32.2 KiB) Exibido 7747 vezes
A figura acima mostra uma fonte luminosa e uma lente convergente, presas a molas idênticas, de massas desprezíveis e relaxadas. A fonte e a lente são colocadas em contato, provocando a mesma elongação nas três molas. Em seguida são soltas e movimentam-se sem atrito. Do instante inicial até o instante em que a fonte e a lente se encontram novamente, determine o tempo total em que a imagem formada é virtual.

Dados:

constante elastica das molas: [tex3]k = 20g/s^2[/tex3]

massa da fonte luminosa + suporte: [tex3]20g[/tex3]

massa da lente: [tex3]10g[/tex3]

elongação das molas no instante do contato: [tex3]10cm[/tex3]

distancia focal da lente: [tex3]26,25cm[/tex3]
Resposta

[tex3]\Delta t=2arccos(\frac{1}{4})+2arccos(\frac{3}{4})[/tex3]

Última edição: LPavaNNN (Sáb 28 Jan, 2017 13:43). Total de 1 vez.


Lucas Pavan

Avatar do usuário
card0z0
iniciante
Mensagens: 8
Registrado em: Qui 29 Jun, 2017 09:07
Última visita: 26-07-17
Jun 2017 29 10:57

Re: V Maratona de Física IME/ITA

Mensagem não lida por card0z0 »

Muito bom essa campanha, Parabéns ao pessoal.
_____________________________________________________________________________
"O mundo da voltas, olhando aqui vejo que a circunferência do giro não confere com a minha :P "

Última edição: card0z0 (Qui 29 Jun, 2017 10:57). Total de 1 vez.



Avatar do usuário
Jvrextrue13
1 - Trainee
Mensagens: 77
Registrado em: Sáb 18 Jul, 2020 16:58
Última visita: 09-06-21
Jul 2020 29 16:30

Re: V Maratona de Física IME/ITA

Mensagem não lida por Jvrextrue13 »

Solução do Problema 21
Primeiro, vamos calcular as equações horárias de cada oscilante.
Fonte de luz:
[tex3]x_F=A.cos(w_F.t)\\A=10cm\rightarrow \text{Elongação da mola no instante inicial}\\w_F=\sqrt{\frac{K}{m}}=\sqrt{\frac{20}{20}}=1rad/s\\x_F=10.cos(t)[/tex3]

Lente Convergente:
[tex3]x_L=A.cos(w_L.t)\\A=10cm\\w_L=\sqrt{\frac{2K}{m}}=\sqrt{\frac{2.20}{10}}=2rad/s\\x_L=10.cos(2t)[/tex3]

Agora a parte de óptica da questão:
[tex3]\frac{1}{f}=\frac{1}{p}+\frac{1}{p'}\rightarrow p'=\frac{f.p}{p-f}>0\rightarrow Virtual\\p-f>0\\p < f[/tex3]
Bom, p representa a distância entra a lente e a fonte luminosa, e essa distância é dada por :
[tex3]p=20-x_F-x_L\\20-10cos(t)-10cos(2t)<26,2510^{-2}\\10(2-cos(t)-cos(2t))<26,25.10^{-2}\\2-cos(t)-cos^2(t)+sen^2(t)<26,25.10^{-1}\\-cos(t)-cos^2(t)+1-cos^2(t)<0,625\\-cos(t)-2cos^2(t)<-0,375\\y=cos(t)\\2y^2+y-\frac{3}{8}>0[/tex3]

Resolvendo essa inequação, encontramos

[tex3]cos(t)<-\frac{3}{4}\\ou\\cos(t)>\frac{1}{4}[/tex3]

Logo,

[tex3]\frac{1}{4}< cos(t)<1\\-1< cos(t)<-\frac{3}{4}[/tex3]
Aqui o que eu consegui tirar de conclusão foi que o sistema fonte-lente irá conjugar uma imagem virtual durante os intervalos de tempo achados e o total será a soma:
[tex3]T_{total}=2arccos\left(\frac{1}{4}\right)+2arccos\left(\frac{3}{4}\right)[/tex3]

Problema 22
IME(2018) A atmosfera densa de um planeta hipotético possui um índice de refração dependente das condições meteorológicas do local, tais como pressão, temperatura e umidade. Considere um modelo no qual a região da atmosfera é formada por [tex3]k+1[/tex3] camadas de índice de refração diferentes,[tex3]n_0, n_1, ... , n_k,[/tex3] de 1 km de altura cada, onde o índice de refração decai 10% a cada quilômetro de aumento na altitude.Considerando somente os efeitos da reflexão e da refração na atmosfera, se um raio luminoso, proveniente de um laser muito potente for disparado da superfície do planeta, formando um ângulo de [tex3]60º[/tex3] com a tangente à superfície, verifique se este raio alcançará o espaço, e, em caso negativo, determine qual será a altitude máxima alcançada pelo raio.
Dados:
-O planeta é esférico de raio [tex3]6370km[/tex3]
-[tex3]log(9)=0,95[/tex3]
-[tex3]log(2)=0,3[/tex3]
-[tex3]k=9[/tex3]
Anexos
21-0.jpg
21-0.jpg (34.66 KiB) Exibido 5946 vezes
Última edição: Jvrextrue13 (Qui 30 Jul, 2020 14:18). Total de 1 vez.


Ensinar/ajudar é uma das melhores formas de fixar o que já foi estudado :D

Avatar do usuário
Pedrolevi120
iniciante
Mensagens: 3
Registrado em: Qui 30 Jul, 2020 12:50
Última visita: 04-07-21
Jul 2020 30 14:40

Re: V Maratona de Física IME/ITA

Mensagem não lida por Pedrolevi120 »

Resolução da questão 22

:arrow:Fatos importantes:

1.Lei dos senos
[tex3]\frac{a}{sen(A)}=\frac{b}{sen(B) }=\frac{c}{sen(C)}[/tex3]
2.Lei de Snell
[tex3]n_1.sen(\theta _1)=n_2.sen(\theta _2)[/tex3]

:arrow: O raio de luz sairá da superfície do planeta formando um ângulo de 60° com a normal, como mostra a figura:
Figura1.png
Figura1.png (21.93 KiB) Exibido 5923 vezes
Sabemos,pela lei de Snell,que:
[tex3]n_o.sen(\theta _o)=n_1.sen(\theta _1)[/tex3]
[tex3]n_0.sen(\theta _0)=0,9n_o.sen(\theta _1)[/tex3]
[tex3]sen(\theta _o)=0,9sen(\theta _1)[/tex3] [tex3]\rightarrow (1)[/tex3]

Sabemos,pela lei dos senos,que:
[tex3]\frac{R+1}{sen\alpha }=\frac{R}{sen\theta _O}\rightarrow (2)[/tex3]

Isolando [tex3]sen\theta _o[/tex3] em (2), substituindo em (1), e por fim isolando [tex3]sen\theta _1[/tex3] ,temos:
[tex3]sen(\theta _1)=\frac{Rsen\alpha }{0,9(R+1)}[/tex3]

Repetindo o mesmo processo para a refração da camada de indice [tex3]n_1[/tex3] para a camada de índice [tex3]n_2[/tex3] , e depois substituindo valor de [tex3]sen\theta _1 [/tex3] já obtido,chegaremos no seguinte resultado:

[tex3]sen\theta _2=\frac{sen\alpha .R}{(R+2).0,9^{2}}[/tex3]

Observando o padrão, podemos generalizar o seno do ângulo do raio refratado com a normal, para K camadas, da seguinte forma:
[tex3]sen(\theta _k)=sen\alpha .(\frac{R}{R+k}).\frac{1}{0,9^{k}}[/tex3]

Agora , É de fundamental importância notar que:
1.O ângulo [tex3]\alpha=30°\rightarrow sen\alpha =0,5[/tex3] .

2.Como k é no máximo 9,e R é 6370km,temos que R+k vale praticamente R.

Agora,como não sabemos em que camada o raio de luz refletirá totalmente, devemos considerar que o seno do ângulo [tex3]\theta _K[/tex3] vale 1,isto é, o raio sairá rasante com a camada em questão, portanto, temos que:
[tex3]1=\frac{1.6370}{2.6370.0,9^{k}}[/tex3]
[tex3]0,9^{k}=2^{-1}[/tex3]
Aplicando log em ambos os lados:
[tex3]k.log(0,9)=-1.log2[/tex3]
[tex3]k.(log9-log10)=-0,3[/tex3]
[tex3]k.(-0,05)=-0,3[/tex3]
[tex3]k=\frac{0,3}{0,05}[/tex3]
[tex3]k=6[/tex3]
Como cada camada tem 1km,e a reflexão total ocorre da 6° para a 7° camada,temos que a altura máxima atingida pelo raio até refletir totalmente mede 7km.



Problema 23

Um recipiente de vidro contendo gás tem uma lente convergente e uma fonte sonora presas a um
suporte (A) que desliza no trilho (B) a velocidade constante. Um feixe laser (C), que ilumina o objeto (D),
forma imagens reais nítidas por duas vezes em (E), separadas por uma diferença de tempo Δt, sendo
que, entre a formação dessas duas imagens, chegam n bips (pulsos sonoros de mesma duração) no
detector (F) e n − 1 bips são emitidos pela fonte sonora. Considerando que o comprimento do recipiente
é L e a distância focal da lente é f, determine a velocidade do som no gás.
questao.png
questao.png (164.63 KiB) Exibido 5923 vezes



Avatar do usuário
Tassandro
5 - Mestre
Mensagens: 1905
Registrado em: Sáb 15 Fev, 2020 17:01
Última visita: 03-10-23
Localização: Teresina, PI.
Jul 2020 30 15:34

Re: V Maratona de Física IME/ITA

Mensagem não lida por Tassandro »

Resolução do problema 23


Pela Equação de Gauss, podemos achar as posições das imagens:
[tex3]\frac 1f=\frac 1p+\frac 1{p'}\implies\frac 1f=\frac{1}{p}+\frac{1}{L-p}\implies\\
p^2-pL+fL=0\tag*{}[/tex3]
Resolvendo, vem que
[tex3]p_1=\frac{L+\sqrt{L^2-4fL}}{2},p_2=\frac{L-\sqrt{L^2-4fL}}{2}\tag*{}[/tex3]
Podemos observar que [tex3]p_1+p_2=L[/tex3] foi assumida como constante. Logo, a velocidade da fonte será dada
[tex3]v_{fonte}=\frac{p_1-p_2}{Δt}=\frac{\sqrt{L^2-4fL}}{Δt}[/tex3]
Pela Equação do Efeito Doppler,
[tex3]f_{obs}=f_{fonte}\(\frac{v_{som}\pm v_{obs}}{v_{som}\pm v_{fonte}}\)\implies\\
\frac{n}{Δt}=\frac{n-1}{Δt}\(\frac{v_{som}}{v_{som}-v_{fonte}}\)\implies \\
v_{som}=nv_{fonte}\therefore \boxed{v_{som}=\frac{n\sqrt{L^2-4fL}}{Δt}}[/tex3]


Problema 24

(ITA 2014) Partindo do repouso, uma bolinha cai verticalmente sobre um plano inclinado de um ângulo θ com relação à horizontal, originando seguidos choques perfeitamente elásticos. Se d é a distância inicial da bolinha ao plano, obtenha, em função de d, n e θ, a distância do ponto do n-ésimo choque em relação ao ponto do primeiro choque.
Última edição: Tassandro (Qui 30 Jul, 2020 15:34). Total de 1 vez.


Dias de luta, dias de glória.

Avatar do usuário
Jvrextrue13
1 - Trainee
Mensagens: 77
Registrado em: Sáb 18 Jul, 2020 16:58
Última visita: 09-06-21
Jul 2020 30 16:30

Re: V Maratona de Física IME/ITA

Mensagem não lida por Jvrextrue13 »

Solução do problema 24

Durante a primeira queda, de uma altura vertical h até o plano ela chegará com uma velocidade:
[tex3]V_o=\sqrt{2.g.h}\\h=\frac{d}{cos(\theta )}[/tex3]

Como as colisões sempre são perfeitamente elásticas, o ângulo de incidência do velocidade é o mesmo do vetor velocidade que "reflete" no plano inclinado,sendo assim os tempos entre as colisões sempre serão os mesmo. Vamos calcular o tempo entre duas colisões.
Decompondo a gravidade no eixos perpendicular e ao longo do plano inclinando tempos:

Na direção perpendicular ao plano inclinado:
[tex3]0=V_o.cos(\theta )-g.cos(\theta ).t_s\\t_{total}=2t_s=\frac{2V_o}{g}\\t_s\rightarrow \text{tempo de altura máxima em relação a superfície inclinada}[/tex3]

Agora, calculando a distância percorrida entre duas colisões n-ésimas:
[tex3]\Delta S=V_o.sen(\theta ).(t_{total}).(n-1)+gsen(\theta )\frac{[(t_{total})(n-1)]^2}{2}\\\Delta S=\frac{2V_o^2}{g}.sen(\theta )(n-1)+g.sen(\theta )\frac{4.V_o^2(n-1)^2}{2g^2}\\\Delta S=\frac{2.V_o^2.sen(\theta )(n-1)+sen(\theta ).2.V_o^2(n-1)^2}{g}\\\Delta S=\frac{2.V_o^2.sen(\theta )(n^2-n)}{g}\\\Delta S=\frac{2.2.g.h.sen(\theta )(n^2-n)}{g}=4hsen(\theta )(n^2-n)\\\Delta S=4d.tan(\theta )(n^2-n)[/tex3]


Problema 25

IME (2018) A figura acima mostra um dispositivo composto por um motor elétrico, cujo eixo se encontra ligado a uma polia ideal de raio [tex3]R[/tex3] , solidária a uma segunda polia de raio [tex3]r[/tex3] , sem deslizamento. Solidário ao segundo eixo há um disco rígido metálico de raio [tex3]r[/tex3] . Em duas extremidades opostas deste disco, foram fixados dois pêndulos compostos idênticos, com fios ideais e esferas homogêneas, de massa [tex3]m[/tex3] . Existe um fio extensível ligando as esferas inferiores, provendo uma força elástica que as mantém na configuração mostrada na figura. Determine, em função de [tex3]g, m, r[/tex3] e [tex3]R[/tex3] :
a) a velocidade angular [tex3]ω[/tex3] do motor elétrico;
b) a força elástica do fio extensível.
Dado:
• aceleração da gravidade: [tex3]g[/tex3]
Anexos
Anotação 2020-07-30 162830.png
Anotação 2020-07-30 162830.png (63.15 KiB) Exibido 5910 vezes
Última edição: Jvrextrue13 (Sex 31 Jul, 2020 08:30). Total de 1 vez.


Ensinar/ajudar é uma das melhores formas de fixar o que já foi estudado :D

Avatar do usuário
Pedrolevi120
iniciante
Mensagens: 3
Registrado em: Qui 30 Jul, 2020 12:50
Última visita: 04-07-21
Jul 2020 31 11:07

Re: V Maratona de Física IME/ITA

Mensagem não lida por Pedrolevi120 »

Solução do problema 25

Como são dois fios, há dois valores de tração T1 e T2,segue a representação:
coiso.png
coiso.png (22.1 KiB) Exibido 5896 vezes
Escrevendo as equações da lei de newton para as duas bolinhas, atentando para os valores da resultante centrípeta:
1.Bolinha de baixo:
[tex3]T_1cos60+Fel=6rmw'^{2}\rightarrow Eq1[/tex3]
e
[tex3]T_1sen60=mg\rightarrow Eq2[/tex3]

2.Bolinha de cima:
[tex3]T_2cos60-T_1cos60=3rmw'^{2}\rightarrow Eq3[/tex3]
e
[tex3]T_2sen60=T_1sen60+mg\rightarrow Eq4[/tex3]

OBS:Os valores dos raios foram descobertos atraves do cosseno de 60 nos [tex3]\Delta AED [/tex3] e [tex3]\Delta CBA[/tex3] e depois somando o raio r da polia.

De Eq1,vem que:
[tex3]T_1=\frac{2mg\sqrt{3}}{3}[/tex3]

Substituindo o valor de [tex3]T_1[/tex3] em Eq4, e isolando [tex3]T_2[/tex3], temos:
[tex3]T_2=\frac{4mg\sqrt{3}}{3}[/tex3]

Substituindo o valor de [tex3]T_2 [/tex3] na Eq3, e isolando w', temos:
[tex3]w'=\frac{1}{3}\sqrt{\frac{g\sqrt{3}}{r}}[/tex3]

Da teoria de movimento circular, sabemos que as velocidades das duas polias de cima são iguais , logo:
[tex3]w'r=wR[/tex3]
[tex3]w=\frac{w'r}{R}[/tex3]
[tex3]w=\frac{r}{3R}\sqrt{\frac{g\sqrt{3}}{r}}[/tex3]

Isolando a Fel em Eq1, temos:
[tex3]Fel=\frac{\sqrt{3}}{3}mg[/tex3]


Problema 26
IME 2010/2011
Anotação 2020-07-31 110505.png
Anotação 2020-07-31 110505.png (157.14 KiB) Exibido 5896 vezes
miau rodo.png
miau rodo.png (40.47 KiB) Exibido 5896 vezes



Avatar do usuário
Tassandro
5 - Mestre
Mensagens: 1905
Registrado em: Sáb 15 Fev, 2020 17:01
Última visita: 03-10-23
Localização: Teresina, PI.
Jul 2020 31 12:54

Re: V Maratona de Física IME/ITA

Mensagem não lida por Tassandro »

Nota: O enunciado deve ser digitado

Resolução do Problema 26
20200731_121329.jpg
20200731_121329.jpg (21.33 KiB) Exibido 5885 vezes
a) É fácil ver que o módulo da velocidade relativa da carga em relação ao espelho vale 2v. Assim, [tex3]|v'|=2v[/tex3]
Completando os ângulos, temos que as velocidades relativas ao espelho valem
[tex3]v'_x=2v\sen(2α-90\degree)=-2vcos2α\\
v'_y=-2v\cos(2α-90\degree)=-2v\sen2α[/tex3]

Para um observador no solo,

[tex3]\boxed{v_x=-2v\cos2α+v\implies v_x=v(1-2\cos2α)\\
v_y=v'y=-2v\sen2α}[/tex3]

b) A aceleração centrípeta vale [tex3]a_{cp}=\frac{v^2}{L}[/tex3] na direção do eixo y, e a aceleração tangencial é causada pela componente horizontal da força elétrica, assim,
[tex3]F_{r_{t}}=ma_t\implies\frac{kQ^2\cosβ}{r^2}=ma_t\implies a_t=\frac{kQ^2\cosβ}{m\(\frac{d}{\senβ}\)^2}\therefore\boxed{a_t=\frac{kQ^2\sen^2β\cosβ}{md^2}}\\[/tex3]
20200731_121910.jpg
20200731_121910.jpg (29.01 KiB) Exibido 5885 vezes
c) Como o espelho está em MRU, a aceleração da imagem é a mesma no referencial do espelho e em um referencial em repouso.
Decompondo, temos que
[tex3]\begin{cases}a_x=a_{cp}\cos(2α-90\degree)+a_t\sen(2α-90\degree)\\
a_y=a_{cp}\sen(2α-90\degree)-a_t\cos(2α-90\degree)\end{cases}\\
\therefore a_x=\frac{v^2}{L}\sen2α-\frac{kQ^2\sen^2β\cosβ\cos2α}{md^2},
a_y=\frac{v^2}{L}\cos2α-\frac{kQ^2\sen^2β\cosβ\sen2α}{md^2}[/tex3]

Problema 27
20200731_125333.jpg
20200731_125333.jpg (6.73 KiB) Exibido 5885 vezes
(IME 2016) Uma corda de comprimento L e densidade linear constante gira em um plano em torno da
extremidade fixa no pontoA a umavelocidade angular constante igual a ω. Um pulso
ondulatório é gerado a partir de uma das extremidades. A velocidade vdo pulso, no referencial da corda, a uma distância r da extremidade fixa é dada por:
a)[tex3]ω\frac{(L-r)}{\sqrt2}[/tex3]
b)[tex3]ω\sqrt\frac{L(L-r)}{2}[/tex3]
c)[tex3]ω\frac{(L^2-r^2)}{\sqrt2L}[/tex3]
d)[tex3]ω\sqrt\frac{L^2-r^2}{2}[/tex3]
e)[tex3]ω\frac{L\sqrt{L-r}}{\sqrt{2(L+r)}}[/tex3]
Resposta

d
Anexos
20200731_121910.jpg
20200731_121910.jpg (29.01 KiB) Exibido 5885 vezes

Última edição: Tassandro (Sex 31 Jul, 2020 12:56). Total de 1 vez.


Dias de luta, dias de glória.

Responder
  • Tópicos Semelhantes
    Respostas
    Exibições
    Última msg

Voltar para “Maratonas de Física”